Question and Answers Forum

All Questions      Topic List

Number Theory Questions

Previous in All Question      Next in All Question      

Previous in Number Theory      Next in Number Theory      

Question Number 130109 by Adel last updated on 22/Jan/21

2+(3/2^3 )+(4/3^3 )+(5/4^3 )+(6/5^3 )+∙∙∙∙∙∙∙∙∙=?

$$\mathrm{2}+\frac{\mathrm{3}}{\mathrm{2}^{\mathrm{3}} }+\frac{\mathrm{4}}{\mathrm{3}^{\mathrm{3}} }+\frac{\mathrm{5}}{\mathrm{4}^{\mathrm{3}} }+\frac{\mathrm{6}}{\mathrm{5}^{\mathrm{3}} }+\centerdot\centerdot\centerdot\centerdot\centerdot\centerdot\centerdot\centerdot\centerdot=? \\ $$

Answered by Olaf last updated on 22/Jan/21

S = Σ_(n=1) ^∞ ((n+1)/n^3 )  S = Σ_(n=1) ^∞ (1/n^2 )+Σ_(n=1) ^∞ (1/n^3 )  S = ζ(2)+ζ(3) = (π^2 /6)+ζ(3)  ξ(3) : Apery constant ≈ 1,202

$$\mathrm{S}\:=\:\underset{{n}=\mathrm{1}} {\overset{\infty} {\sum}}\frac{{n}+\mathrm{1}}{{n}^{\mathrm{3}} } \\ $$$$\mathrm{S}\:=\:\underset{{n}=\mathrm{1}} {\overset{\infty} {\sum}}\frac{\mathrm{1}}{{n}^{\mathrm{2}} }+\underset{{n}=\mathrm{1}} {\overset{\infty} {\sum}}\frac{\mathrm{1}}{{n}^{\mathrm{3}} } \\ $$$$\mathrm{S}\:=\:\zeta\left(\mathrm{2}\right)+\zeta\left(\mathrm{3}\right)\:=\:\frac{\pi^{\mathrm{2}} }{\mathrm{6}}+\zeta\left(\mathrm{3}\right) \\ $$$$\xi\left(\mathrm{3}\right)\::\:\mathrm{Apery}\:\mathrm{constant}\:\approx\:\mathrm{1},\mathrm{202} \\ $$

Commented by Adel last updated on 22/Jan/21

what thes ς(2)  and  ς(3)   ?

$$\mathrm{what}\:\mathrm{thes}\:\varsigma\left(\mathrm{2}\right)\:\:\mathrm{and}\:\:\varsigma\left(\mathrm{3}\right)\:\:\:? \\ $$

Commented by Ar Brandon last updated on 22/Jan/21

ζ(n)=Σ_(k=1) ^∞ (1/k^n )

$$\zeta\left(\mathrm{n}\right)=\underset{\mathrm{k}=\mathrm{1}} {\overset{\infty} {\sum}}\frac{\mathrm{1}}{\mathrm{k}^{\mathrm{n}} } \\ $$

Commented by Olaf last updated on 22/Jan/21

ζ(s) = Σ_(n=1) ^∞ (1/n^s )  ζ(s) = Riemann zeta function.  For some values of s it′s known.  For example ζ(2) = (π^2 /6), ζ(4) = (π^4 /(90))  But for some values of s it′s approximative.  For example ζ(3) = Σ_(n=1) ^∞ (1/n^3 ) ≈ 1,202056903.

$$\zeta\left({s}\right)\:=\:\underset{{n}=\mathrm{1}} {\overset{\infty} {\sum}}\frac{\mathrm{1}}{{n}^{{s}} } \\ $$$$\zeta\left({s}\right)\:=\:\mathrm{Riemann}\:\mathrm{zeta}\:\mathrm{function}. \\ $$$$\mathrm{For}\:\mathrm{some}\:\mathrm{values}\:\mathrm{of}\:{s}\:\mathrm{it}'\mathrm{s}\:\mathrm{known}. \\ $$$$\mathrm{For}\:\mathrm{example}\:\zeta\left(\mathrm{2}\right)\:=\:\frac{\pi^{\mathrm{2}} }{\mathrm{6}},\:\zeta\left(\mathrm{4}\right)\:=\:\frac{\pi^{\mathrm{4}} }{\mathrm{90}} \\ $$$$\mathrm{But}\:\mathrm{for}\:\mathrm{some}\:\mathrm{values}\:\mathrm{of}\:{s}\:\mathrm{it}'\mathrm{s}\:\mathrm{approximative}. \\ $$$$\mathrm{For}\:\mathrm{example}\:\zeta\left(\mathrm{3}\right)\:=\:\underset{{n}=\mathrm{1}} {\overset{\infty} {\sum}}\frac{\mathrm{1}}{{n}^{\mathrm{3}} }\:\approx\:\mathrm{1},\mathrm{202056903}. \\ $$

Commented by talminator2856791 last updated on 22/Jan/21

 is there proof that not all riemann zeta functions    have closed forms?

$$\:\mathrm{is}\:\mathrm{there}\:\mathrm{proof}\:\mathrm{that}\:\mathrm{not}\:\mathrm{all}\:\mathrm{riemann}\:\mathrm{zeta}\:\mathrm{functions}\: \\ $$$$\:\mathrm{have}\:\mathrm{closed}\:\mathrm{forms}? \\ $$

Answered by Ar Brandon last updated on 22/Jan/21

S=Σ_(k=1) ^∞ ((k+1)/k^3 )=Σ_(k=1) ^∞ {(1/k^2 )+(1/k^3 )}     =ζ(2)+ζ(3)

$$\mathrm{S}=\underset{\mathrm{k}=\mathrm{1}} {\overset{\infty} {\sum}}\frac{\mathrm{k}+\mathrm{1}}{\mathrm{k}^{\mathrm{3}} }=\underset{\mathrm{k}=\mathrm{1}} {\overset{\infty} {\sum}}\left\{\frac{\mathrm{1}}{\mathrm{k}^{\mathrm{2}} }+\frac{\mathrm{1}}{\mathrm{k}^{\mathrm{3}} }\right\} \\ $$$$\:\:\:=\zeta\left(\mathrm{2}\right)+\zeta\left(\mathrm{3}\right) \\ $$

Answered by Dwaipayan Shikari last updated on 22/Jan/21

Approximation of Σ_(n=1) ^∞ (1/n^3 )  f(x)=(1/x^3 )  Σ_(n=1) ^∞ (1/n^3 )=lim_(z→∞) ∫_1 ^∞ (1/x^3 )+((f(z)+f(1))/2)+Σ_(n=1) ^∞ (B_(2n) /((2n)!))(f^(2k−1) (z)−f^(2k−1) (1))  =(1/2)+(1/2)+(((3B_2 )/(2!))+((3.4.5)/(4!))B_4 +((3.4.5.6.7)/(6!))B_6 +((3.4.5.6.7.8.9)/(8!))B_8 +...)  =1.2020....   (B_n =Bernoulli number)  Even zeta function ζ(2n)  (n∈Z)  ζ(2)=(π^2 /6) ,ζ(4)=(π^4 /(90)),ζ(6)=(π^6 /(945)) ,ζ(8)=(π^8 /(9450))  ,ζ(10)=(π^(10) /(93555))....

$${Approximation}\:{of}\:\underset{{n}=\mathrm{1}} {\overset{\infty} {\sum}}\frac{\mathrm{1}}{{n}^{\mathrm{3}} } \\ $$$${f}\left({x}\right)=\frac{\mathrm{1}}{{x}^{\mathrm{3}} } \\ $$$$\underset{{n}=\mathrm{1}} {\overset{\infty} {\sum}}\frac{\mathrm{1}}{{n}^{\mathrm{3}} }=\underset{{z}\rightarrow\infty} {\mathrm{lim}}\int_{\mathrm{1}} ^{\infty} \frac{\mathrm{1}}{{x}^{\mathrm{3}} }+\frac{{f}\left({z}\right)+{f}\left(\mathrm{1}\right)}{\mathrm{2}}+\underset{{n}=\mathrm{1}} {\overset{\infty} {\sum}}\frac{{B}_{\mathrm{2}{n}} }{\left(\mathrm{2}{n}\right)!}\left({f}^{\mathrm{2}{k}−\mathrm{1}} \left({z}\right)−{f}^{\mathrm{2}{k}−\mathrm{1}} \left(\mathrm{1}\right)\right) \\ $$$$=\frac{\mathrm{1}}{\mathrm{2}}+\frac{\mathrm{1}}{\mathrm{2}}+\left(\frac{\mathrm{3}{B}_{\mathrm{2}} }{\mathrm{2}!}+\frac{\mathrm{3}.\mathrm{4}.\mathrm{5}}{\mathrm{4}!}{B}_{\mathrm{4}} +\frac{\mathrm{3}.\mathrm{4}.\mathrm{5}.\mathrm{6}.\mathrm{7}}{\mathrm{6}!}{B}_{\mathrm{6}} +\frac{\mathrm{3}.\mathrm{4}.\mathrm{5}.\mathrm{6}.\mathrm{7}.\mathrm{8}.\mathrm{9}}{\mathrm{8}!}{B}_{\mathrm{8}} +...\right) \\ $$$$=\mathrm{1}.\mathrm{2020}....\:\:\:\left({B}_{{n}} ={Bernoulli}\:{number}\right) \\ $$$${Even}\:{zeta}\:{function}\:\zeta\left(\mathrm{2}{n}\right)\:\:\left({n}\in\mathbb{Z}\right) \\ $$$$\zeta\left(\mathrm{2}\right)=\frac{\pi^{\mathrm{2}} }{\mathrm{6}}\:,\zeta\left(\mathrm{4}\right)=\frac{\pi^{\mathrm{4}} }{\mathrm{90}},\zeta\left(\mathrm{6}\right)=\frac{\pi^{\mathrm{6}} }{\mathrm{945}}\:,\zeta\left(\mathrm{8}\right)=\frac{\pi^{\mathrm{8}} }{\mathrm{9450}}\:\:,\zeta\left(\mathrm{10}\right)=\frac{\pi^{\mathrm{10}} }{\mathrm{93555}}.... \\ $$

Commented by Adel last updated on 23/Jan/21

thanks bro

$$\mathrm{thanks}\:\mathrm{bro} \\ $$

Terms of Service

Privacy Policy

Contact: info@tinkutara.com